Solve using substitution.
X = 1
6x + y = 1

please help

Answers

Answer 1

Answer:   (1,-5)

In other words, x = 1 and y = -5

=====================================================

Work Shown:

6x + y = 1

6(1) + y = 1

6 + y = 1

y = 1-6

y = -5

The solution as an ordered pair is (1,-5)

Notice I started with the second equation and substituted in the first equation. The idea is to somehow reduce the two variables into one variable, so you can solve for that remaining variable.

Answer 2

Answer:

6x+y=1

or,6×1+y=1

or,6+y=1

or,y=6-1

ans,y=5


Related Questions



Which term has the definition "A relationship in which two variables change at the same rate

Answers

positive correlation

(i believe thats what it is)

PLEASE HELP ME WITH GEOMETRY

Answers

Answer:

The answer is A.

Step-by-step explanation:

It is A because both sides match up except for the last pair, where they are the same. Hope this helped!

The perimeter of a rectangular floor is 240 feet. Find the dimensions of the floor if the length is three times the width
The floor has a length of feet and a width of feet.

Answers

Answer:

  length: 90 ft

  width: 30 ft

Step-by-step explanation:

The sum of length and width is half the perimeter, so is 120 ft. The length is 3/(3+1) = 3/4 of the sum, so is (3/4)(120 ft) = 90 ft. The width is 120 ft -90 ft = 30 ft.

The floor has a length of 90 feet and a width of 30 feet.

Which statement is true about the function f(x)= Vx?
O The domain of the graph is all real numbers.
O The range of the graph is all real numbers.
O The domain of the graph is all real numbers less than or equal to 0.
O The range of the graph is all real numbers greater than or equal to 0.

Answers

Answer:

The statement "the range of the graph is all real numbers greater than or equal to 0" is true.

Step-by-step explanation:

As the function is given by

                                             

As we know that the domain is the set of all possible input values or all  x -values that a function passes through.  In other words, domain consists of all the possible input values shown on the x-axis.

Range is the set of all y-values that a function passes through. In other words, range consists of all the possible output values shown on the y-axis.

For the given function

                                       

The domain of the given function  

The domain of  can also be written as: {x | x ≥ 0}

It means the domain of the graph of this function is all real number greater than or equal to 0.

The domain of the graph of this function can not be less than zero, because if we put any negative real number in this function, it would make the the function undefined.

Please check the attached graph as shown in figure a, to visualize the domain and range of  

From the graph.

it is visible that the range of the given function  

The range of  can also be written as: {y | y ≥ 0}

In other words, for every input value in the domain of this function, the range of   will be all real numbers greater than or equal to 0. Please see the attached graph as shown in figure a.

So, from the entire discussion, we can safely say that the statement "the range of the graph is all real numbers greater than or equal to 0" is true.

What is is b? 2-b/3 =-5/2

Please explain thoroughly

Answers

Answer:

I'm not sure you mean 2-(b/3) = -5/2 or (2-b)/3 = -5/2

Step-by-step explanation:

if

2-(b/3) = -5/2

12-2b = -15

12+15 = 2b

27 = 2b

27/2 = b

else

(2-b)/3 = -5/2

2(2-b) = -5×3

4-2b = -15

4+15 = 2b

19 = 2b

19/2 = b

Given 2 angles that measure 50 and 80 and a side that measure 4 feet how many triangles if any can be condctuted

Answers

Answer:

The number of unique triangles that can be constructed from the given values is; Only one triangle

Step-by-step explanation:

We are given the 2 angles of a triangle as;  

∠1 = 50°  

∠2 = 80°  

Now, we know that sum of angles in a triangle is 180°. This means that if the third angle is denoted as ∠3, then we have;  

∠1 + ∠2 + ∠3 = 180°  

Thus;  

∠3 = 180 - (∠1 + ∠2)  

∠3 = 180 - (50 + 80)  

∠3 = 180 - 130  

∠3 = 50°  

Thus; ∠1 = ∠3 = 50°  

A triangle with two equal angles is called an isosceles triangle. Which means that it will also have 2 of its' sides to be equal.  

Thus, in conclusion, only one unique triangle can be drawn.


An article when sold at a profit of 5% yields Rs 50 more than when sold at a loss of 5%. What was the cost price of the article?​

Answers

Answer:

  Rs 500

Step-by-step explanation:

For cost c, the price that gives a 5% profit is ...

  c + 5%·c = 1.05c

The price that gives a 5% loss is ...

  c -5%·c = 0.95c

The difference of these prices is Rs 50:

  1.05c -0.95c = 50

  0.10c = 50 . . . . simplify

  c = 500 . . . . . . . divide by 0.1

The cost price of the article is Rs 500.

Find the area and perimeter of the shape given below.

Answers

Answer:

perimeter: 14+5+6+6+8+10=59

area: 14x5x6x10=4200

perimeter means adding all the sides including the ones that arent labeled, and the area is multiplying all the numbers included.

Dev has a monthly food budget of $182. He maps the amount of money,x, he spends each month to the number of food items he buys. What are the constrains on the domain?

Answers

Using it's concept, it is found that the domain of the function is [tex]0 \leq x \leq 182[/tex], that is, the constraints are:

x is of at least $0.x is of at most $182.

The domain of a function is the set that contains all possible values for the functions.

In this problem, the input of the function is how much he spends on food.

His budget is of $182, hence, considering this and the fact that he cannot spend a negative amount on food, the constraints are that x is between $0 and his budget of $182(inclusive), and the domain is [tex]0 \leq x \leq 182[/tex].

To learn more about the domain of functions, you can take a look at https://brainly.com/question/25897115

The domain of a function is the set of input values the function can have.

The constraints on the domain are: the amount (x) is at least $0, and it is at most $182

From the question, Dev's budget on food is given as:

[tex]Budget = \$182[/tex]

This means that, he cannot spend more than $182.

Represent the amount with x.

So, we have:

[tex]x \le 182[/tex]

Also, he cannot spend a negative amount.

So, we have:

[tex]x \ge 0[/tex]

So, the domain is:

[tex]x \ge 0[/tex] and [tex]x \le 182[/tex]

Rewrite as:

[tex]0 \le x[/tex] and [tex]x \le 182[/tex]

Combine both inequalities

[tex]0 \le x \le 182[/tex]

Hence, the constraints on the domain are: the amount (x) is at least $0, and it is at most $182

Read more about domain at:

https://brainly.com/question/10197594

can someone please help me with this and the first to answer will get marked as brainiest also i need the work to be shown cause i know the answer but just not the work cause i’m confused pls help

Answers

Answer:

x = 3

Step-by-step explanation:

a triangle has 180 degrees in all

a square for an angle is 90 degrees

we can add them all together and get 180

90 + 45 + 8x + 21 = 180

combine like terms

156 + 8x = 180

subtract 156 to each side

8x = 24

then find x

x = 3

Answer:

x=3

Step-by-step explanation:

So the sum of angles of triangle is always 180 degrees.

We already have 45 and 90 but 8x+21 is unknown.

We can solve this by making an equation which is

45 + 90 + (8x+21) = 180.

If we solve this it will be

8x+21+135=180

8x+156 (-156) =180 (-156)

8x (/8) = 24 (/8)

x=3

Which inequality represents this statement?

The product of −6 and a number is no less than −12.


−6n≤−12

−6n>−12

−6n≥−12

−6n<−12

Answers

Answer:

−6n≥−12

Step-by-step explanation:

The correct inequality to show the given statement is,

⇒ - 6n ≥ - 12

Where, 'n' is any number.

What is Inequality?

A relation by which we can compare two or more mathematical expression is called an inequality.

Given that;

The statement is,

''The product of - 6 and a number is no less than - 12.''

Now, Let a number = n

So, We can formulate the correct inequality is,

⇒ - 6n ≥ - 12

Learn more about the inequality visit:

https://brainly.com/question/25944814

#SPJ2

Solve for x:
4x-3Divided by 2 =5/3

Answers

Answer:

x=19/12

Step-by-step explanation:

(4x-3)/2 = 5/3

4x-3 = 10/3

3 = 9/3

4x = 10/3 + 9/3 = 19/3

x = 19/12

Hey there!

4x - 3/2 = 5/3

ADD 3/2 to BOTH SIDES

4x - 3/2 + 3/2 = 5/3 + 3/2

CANCEL out: -3/2 + 3/2 because it gives you 1

KEEP: 5/3 + 3/2 because it helps solve for the x-value.

NEW EQUATION: 4x = 5/3 + 3/2

SIMPLIFY IT!

4x = 19/6

DIVIDE 4 to BOTH SIDES

4x ÷ 4 = 19/6 ÷ 4

CANCEL out: 4 ÷ 4 because it gives you 1

KEEP: 19/6 ÷ 4 because it helps solve for the x-value

NEW EQUATION: x = 19/6 ÷ 4

SIMPLIFY IT!

x = 19/24

Good luck on your assignment and enjoy your day!

~Amphitrite1040:)

I need some urgent help please

Answers

Answer:

124°

Step-by-step explanation:

180°( n - 2 )

180° ( 5 - 2 ) = 540°

( 2x - 3 )° + ( 3x - 2 )° + 2( 3x - 4 )° + ( 2x + 7 )° = 540°

( 2x + 3x + 6x + 2x ) + ( - 3 - 2 - 8 + 7 ) = 540

13x - 6 = 540

13x = 546

x = 42

3(42) - 2 = 124°

Find two numbers the quotient is between . Then estimate quotient. 41 divided by 3

Answers

Answer:

13.6

Step-by-step explanation:

The quotient is the outcome of a division problem so it is what 41 divided by  3 equals. Which it equals 13.666 in short 13.6

The first three terms of a geometric sequence are as follows.
-10, -30, -90
Find the next two terms of this sequence.
Give exact values (not decimal approximations)

Answers

Answer:

-270,810

Step-by-step explanation:

c.r.  r=-30/-10=3

[tex]a_{n} =a_{1}r^{n-1} \\a_{4}=-10(3)^{4-1}\\=-10(3)^3=-10 \times 27=-270\\a_{5}=-10 \times(3)^{5-1}\\=10 \times3^4\\=-810[/tex]

How can you identify the number of solutions of linear equations?

Answers

Answer:

A system of two equations can be classified as follows: If the slopes are the same but the y-intercepts are different, the system has no solution. If the slopes are different, the system has one solution. If the slopes are the same and the y-intercepts are the same, the system has infinitely many solutions.

Mark me as brainlist plz :)

V is themidpoint of UW. If UV = x + 8 and VW = 9x, what is VW?

Answers

Answer:

x=1

Step-by-step explanation:

If V is the midpoint of the line UW, that would mean that there's an equal distance between UV and 9x. So, the value of UV and VW would eed to be the same.

x+8=9x

8=8x

x=1

Determine the slope from the given graph below:

A ⇹ The slope is -7
B ⇹ The slope is−16
C ⇹ The slope is 6
D ⇹ The slope is -6

Answers

Step-by-step explanation:

although there is no graph visible here, I suspect this is the same question as I just answered twice already.

the slope is -6.

D is the right answer.

Can you simplify these two?
5 points for each.

Answers

Answer:

See attached files.

The answers are there.

help plzzzzzzzzzzzzzzzzzzzzz

Answers

Answer:

ang hirap naman plss answee

in a survey of 500 voters 430 said they would vote for the same candidate again what percent of the votes would vote the same way again

Answers

86% would vote for the same person again

Given the following acceleration function of an object moving along a line, find the position function with the given initial velocity and position.
a(t) = -24; v(0) = 17, s(0) = 0
s(t) =

Answers

Answer:

Position function is [tex]s(t)=-12t^2[/tex]

Step-by-step explanation:

Given [tex]a(t)=-24[/tex], since velocity is the antiderivative of acceleration, then [tex]v(t)=-24t[/tex].

Also, since position is the antiderivative of velocity, then [tex]s(t)=-12t^2[/tex].

lf the exchange rate of 1 dollar is Rs 72.45. how many dollars exchanged for Rs 1449 ?

Answers

Step-by-step explanation:

solution

here,

1 dollar = rs 72.45

or, 1/72.45 dollars= rs 1

or, rs 1 = 1/72.45 dollars

now,

rs 1449= 1449/72.45 dollars

or,rs1449= 20 dollars

therefore, 20 dollars can be exchanged for rs 1449

hey guys I could really use some help on this.

Answers

Answer:

y = - [tex]\frac{10}{3}[/tex] x - 23

Step-by-step explanation:

The equation of a line in slope- intercept form is

y = mx + c ( m is the slope and c the y- intercept )

Calculate m using the slope formula

m = [tex]\frac{y_{2}-y_{1} }{x_{2}-x_{1} }[/tex]

with (x₁, y₁ ) = (- 9, 7 ) and (x₂, y₂ ) = (- 6, - 3 )

m = [tex]\frac{-3-7}{-6-(-9)}[/tex] = [tex]\frac{-10}{-6+9}[/tex] = [tex]\frac{-10}{3}[/tex] = - [tex]\frac{10}{3}[/tex] , then

y = - [tex]\frac{10}{3}[/tex] x + c ← is the partial equation

To find c substitute either of the 2 points into the partial equation

Using (- 6, - 3 ) , then

- 3 = 20 + c ⇒ c = - 3 - 20 = - 23

y = - [tex]\frac{10}{3}[/tex] x - 23 ← equation of line

Answer:

y= -10x/3 + 1

Step-by-step explanation:

gradient of a line= (Y2-Y1)/(X2-X1)

= (-3-7)/[-6-(-9)]

=-10/3

equation of a line

(Y-Y1)/(X-X1)= gradient

(Y-7)/(X+9)= -10/3

3(Y-7)= -1d0(X+9)

3y-21= -10x -19

3y= -10x + 3

dividing both sides by 3

y= -10x/3 + 1

In a newspaper, it was reported that the number of yearly robberies in Springfield in 2011 was 250, and then went up by 20% in 2012. How many robberies were there in Springfield in 2012?

Answers

First, we need to find out what 20% of 250 is

250 x .2 = 50

You then add the answer (50) to the yearly number of robberies in Springfield in 2011 (250)

250 + 50 = 300

The number of robberies in Springfield in 2012 is 300

pls help me
I am giving brainliest​

Answers

Solution

1/49 x (441x^2 - 16)

Answer is 1/49 x (21x - 4) x (21x + 4)

Write the word sentence as an equation. Then solve.

The sum of a number a and 12 is -6.

Answers

Answer:

a + 12 = -6 , a = -18

Step-by-step explanation:

Hi there!

the first half of the statement says “the *sum* of a number a and 12, so we can write that as a+12. The second half of the statement says that the sum of both of those values is -6, meaning that a + 12 = -6. Knowing this, we can now solve. In order to isolate a, subtract 12 from both sides of the equation. You will be left with a = -18.

Hope this helps!

45
899
769
459
X?
Use the exterior angle sum to find the value of x in the diagram
above.
A.72
B.105
C.108
D.435

Answers

Answer:d

Step-by-step explanation:

Answer:

d

Step-by-step explanation:

[(1 1/5−2/5)⋅(−3/4)3]÷(−9)

Answers

Given that:

[{1(1/5)−(2/5)⋅(−3/4)3}]÷(−9)

= [{(5+1)/5 - (2/5).(-3/4)3}]÷(-9)

= [{(6/5) - (2/5).(-3/4)3}]÷(-9)

= [{(6/5) - (2/5).(-3/4)3}]÷(-9)

= [{(6/5) - (2/5).(-3*3)/4}]÷(-9)

= [{(6/5) - (2/5) . (-9/4)}]÷(-9)

= [{(6/5) - (2/5) × (-9/4)}]÷(-9)

= [{{6/5) - {{2*(-9)}/{5*4}]}]÷(-9)

= [{(6/5) - {-9/(5*2)}}]÷(-9)

= [{(6/5) - (-9/10)}]÷(-9)

Take the LCM of 5 & 10 is 10.

= [{(6*2 + 9*1)/10}]÷(-9)

= [{(12+9)/10}]÷(-9)

= [{-(21/10)}]÷(-9)

= -(21/10) ÷ (-9)/1

= -(21/10) × (1/9)

= {(-21*1)/(10*9)]

= {-21/90)

= {(-21÷3)/(90÷3)}

= -9/30

= {(-9÷3)/(30÷3)}

= (-3/10) = -3/10 Ans.

What is the gradient?

Answers

Answer:

The gradient is negative.

Step-by-step explanation:

The gradient represents the rise/run ratio. Always read the line's path left to right, and if it goes up, then it is positive, down means negative gradient, and if it doesnt move then it 0. It is undefined if the line is vertical.

Other Questions
Please anyone help me How can you build your resilience to help counter risk factors for bullying 23.What would happen to the substance in the jar if energy was added? How does a forest fire impact a population of birds that nest in the trees. i need help asapWhat is the meaning of the word blanket in the following sentence: Douglas fir, aspen, and pion pine trees in the Cibola National Forest blanket the mountain. Spires of volcanic rock rise on the north slope.? A. warm up B. hide C. cover D. color alguien podria ayudarme? PLEASE HELP ASAP!!!! (> Which is a possible food chain in the ocean from start to finish?a.phytoplankton, zooplankton, small fish, large fish, large sharkb.phytoplankton, large shark, zooplankton, large fish, small fishc.large shark, large fish, small fish, zooplankton, phytoplanktond.zooplankton, phytoplankton, large fish, small fish, large shark in _________, enzymes cut DNA into fragments, which are separated by size to form a pattern of bands.a. selective breedingb. cloning c. dna fingerprinting d. protein synthesis Explain the Exam for Edge 50 pointsis it screen recorded or can you use brainly I'm stressed because if I can't use brainly I am 99.9% sure that I will fail:'( If you want to eat vegetables with high levels of nutrients, you should look for vegetables that have .. A bright colors. B large seeds or pits. a strong scent. D leaves and roots attached. How did the gilded age wealthy class ( captains of industry) exploit the poor and immigrant class of workers for their benefits and profit? What was Alexander Hamilton's IQ? HELP QUICK PLS Which property is used in the following expression? (4 + 8) + 6 = 4 + (8 + 6) Associatve Property Commutitive Property Distributive property what percentage is 879.75 of 900 Look at triangle ABC. Coordinate grid shows negative 5 to positive 5 on the x axis and y axis at intervals of 1. A triangle ABC is shown with A at ordered pair 4, 5, B at ordered pair 2, 2, and C at ordered pair 4, 2. What is the length of side AB of the triangle? (1 point) a 3 b 5 c Square root of 6 d Square root of 13 The graphic titled "Where They're Living Solo" suggests that the practice of living alone is -adominantbglobaltemporarycyclicalOd What are the parents points for the equation f(x) = 3^x? Suppose your family lives in Muntinlupa City (one of the West Valley Fault Line area) and decided to move in to the work place of your father in Teresa, Rizal which is approximately 50 km apart, if renting a van costs an initial amount of 4000php for maximum of 30km and an additional payment of 200php for every 5km, how much will your father pay in renting a van from Muntinlupa City to Teresa, Rizal? {attach only 1 picture of your table/answer} The Torah contains the sacred writings of what religion?